Skip to main content

Trigonometric Functions and Applications

  • Chapter
  • First Online:
From Calculus to Analysis
  • 4421 Accesses

Abstract

We investigate the sine and cosine functions, show the Weierstrass function is continuous and nowhere differentiable, construct the number π, establish that π is an irrational number, and give a brief treatment of polar coordinates.

This is a preview of subscription content, log in via an institution to check access.

Access this chapter

Chapter
USD 29.95
Price excludes VAT (USA)
  • Available as PDF
  • Read on any device
  • Instant download
  • Own it forever
eBook
USD 39.99
Price excludes VAT (USA)
  • Available as EPUB and PDF
  • Read on any device
  • Instant download
  • Own it forever
Hardcover Book
USD 79.99
Price excludes VAT (USA)
  • Durable hardcover edition
  • Dispatched in 3 to 5 business days
  • Free shipping worldwide - see info

Tax calculation will be finalised at checkout

Purchases are for personal use only

Institutional subscriptions

Author information

Authors and Affiliations

Authors

Corresponding author

Correspondence to Steen Pedersen .

Appendices

Problems

1.1 Problems for Sect. 11.1

  1. 1.

    Let \(f(x):=\log \left (1+x\right )\) for \(x>-1.\) Let

    $$ T\left(x\right):=\sum_{k=0}^{\infty}\frac{f^{\left(k\right)}\left(0\right)}{k!}x^{k} $$

    be the Taylor series of \(f.\) Use the series to define a function

    $$ g\left(z\right):=\sum_{k=0}^{\infty}\frac{f^{\left(k\right)}\left(0\right)}{k!}z^{k} $$

    of a complex variable \(z.\)

    1. a.

      Find the radius of convergence \(R\) of the power series.

    2. b.

      Show \(f(x)=T\left (x\right )\) for \(-R<x<R.\)

    3. c.

      Conclude \(f(x)=g(x)\) for \(-R<x<R.\)

1.2 Problems for Sect. 11.2

  1. 1.

    Prove that \(\int _{0}^{\infty }\frac {sin(xt)}{1+t^{2}}\, dt\) converges uniformly. That is prove the improper integral exists and

    $$ \forall\varepsilon,\exists M,\forall R,R>M\implies\left[\forall x,\int_{0}^{\infty}\frac{\sin(xt)}{1+t^{2}}\, dt-\int_{0}^{R}\frac{\sin(xt)}{1+t^{2}}\, dt<\varepsilon\right]. $$
  2. 2.

    Prove \(e^{\pi }-\pi \neq 20.\)

  3. 3.

    Suppose \(f\) is continuous. Let \(g(x):=\int _{0}^{x}\sin (x-t)f(t)dt.\) Prove \(g''+g=f.\) [Hint: Begin by showing that \(g'\) exists and finding a formula for \(g'.\)]

  4. 4.

    Let \(f(x):=\frac {\cos (x)}{1+x^{2}}.\) Prove \(\int _{0}^{1}f<1.\)

1.3 Problems for Sect. 11.3

  1. 1.

    Fix \(0\leq a<b\leq 2\pi .\) Find the length of the curve \(\phi (t):=e^{it},\) \(t\in \left [a,b\right ].\)

1.4 Problems for Sect. 11.4

  1. 1.

    Does \(\left (\sum _{n=0}^{N}b^{n}\cos (a^{n}x\pi )\right )'\) converge as \(N\to \infty \) for all \(x\)?

  2. 2.

    Use Corollary 9.2.9. to find a sufficient condition for the Weierstrass function to be differentiable.

  3. 3.

    If \(x_{0}=0,\) the proof in the text simplifies. Write out this simplification. [For example, \(\beta _{m}=1\) for all \(m\) and \(\cos \left (a^{n}x_{0}\pi \right )=1.\)]

  4. 4.

    Continuation of Problem 3. Find integers \(\beta _{m}\) such that

    $$ (-1)^{\beta_{m}}\frac{f\left(\frac{\beta_{m}}{a^{m}}\right)-f\left(0\right)}{\frac{\beta_{m}}{a_{m}}-0}\to-\infty. $$

    [Hint: Equation (11.12) must be replaced.]

  5. 5.

    Show it is possible to choose \(\beta _{m}\) such that \(\beta _{m}/a^{m}\to x_{0}\) and

    $$ (-1)^{\beta_{m}}\frac{f\left(\frac{\beta_{m}}{a^{m}}\right)-f\left(x_{0}\right)}{\frac{\beta_{m}}{a_{m}}-x_{0}}\to-\infty. $$

    Thus, \(f'(x_{0})\) cannot be a finite or even an infinite value.

1.5 Problems for Sect. 11.5

An alternative proof that \(\pi \) is irrational is outlined as sequence of problems below. This proof is due to Ivan Morton Niven (25 October 1915, Vancouver to 9 May 1999, Eugene).

Assume \(\pi =a/b\) for some positive integers \(a\) and \(b.\) For any natural number \(n\geq 1,\) let

$$ f_{n}(x):=\frac{x^{n}(a-bx)^{n}}{n!}\text{ and }F_{n}(x):=\sum_{j=0}^{n}(-1)^{j}f_{n}^{(2j)}(x), $$

where \(f^{(k)}\) is the \(k\)th derivative of \(f.\) Note that \(a-b\pi =0.\) Expanding the product in the definition of \(f_{n}\) we see that \(f_{n}(x)=\frac {1}{n!}\sum _{k=n}^{2n}c_{k}x^{k}\) for some integers \(c_{k}.\) Differentiating this polynomial we get

$$ (**)\quad f_{n}^{(j)}(x)=\frac{1}{n!}\sum_{k=\max\{j,n\}}^{2n}c_{k}\frac{k!}{(k-j)!}x^{k-j}, $$

for \(0\leq j\leq 2n.\)

The proof is now completed in 11 easy steps:

  1. 1.

    \(0\leq f_{n}(x)\leq \pi ^{n}a^{n}/n!\) for \(0\leq x\leq \pi \) and all \(N\) [Directly from the definition of \(F_{n}.\)]

  2. 2.

    \(0<\int _{0}^{\pi }f_{n}\sin \) for all \(n.\) [Since both \(f_{n}\) and \(\sin \) are \(>0\) on \(]0,\pi [.\)]

  3. 3.

    There is an \(N\) such that \(\int _{0}^{\pi }f_{n}\sin <1.\) [Is a consequence of 1]

  4. 4.

    \(\frac {k!}{n!(k-j)!}\) is an integer for all \(n\leq j\leq k.\) [\(\frac {k!}{n!(k-n)!}\) is a binomial coefficient, hence an integer. \(\frac {k!}{n!(k-(n+1))!}=(k-n)\frac {k!}{n!(k-n)!},\) etc.]

  5. 5.

    \(f_{n}(x)=f_{n}(\pi -x)\) for all \(x\) and all \(n.\) [Directly from the definition of \(F_{n},\) since \(a-b(\pi -x)=x.\)]

  6. 6.

    \(f_{n}^{j}(0)=f_{n}^{(j)}(\pi )=0,\) for all \(n,j\) such that \(0\leq j<n\). [By 5 and \((**).\)]

  7. 7.

    \(f_{n}^{(j)}(0)\) and \(f_{n}^{(j)}(\pi )\) are integers, for all \(n,j\) such that \(n\leq j\leq 2n.\) [By 4, 5, and \((**).\)]

  8. 8.

    \(F_{n}(0)\) and \(F_{n}(\pi )\) are integers for all \(n.\) [By the definition of \(F_{n},\) 6, and 7]

  9. 9.

    \(F_{n}+F_{n}''=f_{n}\) for all \(n.\) [The point of the \((-1)^{j}\) in the definition of \(F_{n}.\)]

  10. 10.

    \((F_{n}'\sin -F_{n}\cos )'=f_{n}\sin \) for all \(n.\) [Derived using 9]

  11. 11.

    \(\int _{0}^{\pi }f_{n}\sin =F_{n}(0)-F_{n}(\pi )\) is an integer for all \(n.\) [By 8 and 10]

By 2, 3, and 11 \(\int _{0}^{\pi }f_{N}\sin \) is an integer in \(]0,1[.\) This contradiction completes the proof that \(\pi \) is irrational.

Solutions and Hints for the Exercises

Exercise 11.1.1. The ratio test show that the series is absolutely convergent for all \(z.\)

Exercise 11.2.1. \(e^{-iy}=\overline {e^{iy}.}\)

Exercise 11.2.2. \(\sum _{k=0}^{\infty }\frac {(iy)^{k}}{k!}=\cdots +i\cdots .\)

Exercise 11.2.4. Similar to our proof of (11.3).

Exercise 11.2.6. \(\sqrt {6-2\sqrt {3}}<\sqrt {3}\) \(\,\Leftarrow \,\) \(6-2\sqrt {3}<3\) \(\,\Leftarrow \,\) \(3<2\sqrt {3}\) \(\,\Leftarrow \,\) \(9<12.\)

Exercise 11.2.8. \(1=\cos ^{2}(\pi /2)+\sin ^{2}(\pi /2)=0+\sin ^{2}(\pi /2)\) and \(\sin (\pi /2)>0.\)

Exercise 11.2.9. \(\sin (2\pi )=2\sin (\pi )\cos (\pi )=2\cdot 0\cdot (-1)=0\) and \(\cos ^{2}(2\pi )=\cos ^{2}(\pi )-\sin ^{2}(\pi )=\left (-1\right )^{2}-0^{2}=1.\)

Exercise 11.2.10. Similar to \(\cos (x+2\pi )=\cos (x).\)

Exercise 11.2.11. Similar to \(\cos (x+2\pi )=\cos (x)\) and \(\sin (x+2\pi )=\sin (x).\)

Exercise 11.2.12. Use Exercise 11.2.11.

Rights and permissions

Reprints and permissions

Copyright information

© 2015 Springer International Publishing Switzerland

About this chapter

Cite this chapter

Pedersen, S. (2015). Trigonometric Functions and Applications. In: From Calculus to Analysis. Springer, Cham. https://doi.org/10.1007/978-3-319-13641-7_11

Download citation

Publish with us

Policies and ethics